16 mi c. john started a carpool with his coworkers to save money. he and his three passengers split the cost of the toll. if each person pays about $0.81 , which includes their contribution to the toll lane entry fee, how many miles do they travel on the toll lane?

Answers

Answer 1

John and his three passengers travel a total of 20.25 miles on the toll lane.

To solve this problem, we can use the fact that each person pays about $0.81, which includes their contribution to the toll lane entry fee. This means that the total amount of money paid by John and his three passengers is 4 times $0.81, or $3.24.

We can then use this information to find the cost per mile of the toll lane. If they traveled a total of x miles on the toll lane, then the cost per mile would be:

$3.24 / x

We can set this equal to the given cost of 16 cents per mile:

$0.16 = $3.24 / x

Multiplying both sides by x, we get:

x * $0.16 = $3.24

Dividing both sides by $0.16, we get:

x = $3.24 / $0.16

x = 20.25 miles

In summary, to find the distance they traveled on the toll lane, we used the fact that they split the cost of the toll, and that each person paid about $0.81. We then set the cost per mile equal to the given cost of 16 cents per mile, and solved for the distance traveled on the toll lane, which turned out to be 20.25 miles.

To learn more about miles click on,

https://brainly.com/question/30298989

#SPJ4


Related Questions

Footy. You play in an inter- school footy competition. Curiously, in one of the rounds the total number of points scored by each team is the same, so that all games are not only drawn, but also have the same final score. In that same round your team scored 1/13th of all goals and 1/15th of all behinds. How many teams play in the competition?

Answers

There are 195 games played in the competition.

Let the total number of points scored in each game be represented by the variable "x". Since a goal is worth 6 points and a behind is worth 1 point, we can write an equation in terms of "x":

6a/13 + b/15 = x

where "a" is the total number of goals scored and "b" is the total number of behinds scored by your team in the round.

Since all games have the same final score, we know that the total number of points scored in the round is equal to the number of games played times the final score:

x * number of games = total points scored

We also know that the total number of points scored in the round is equal to the total number of goals scored (by all teams) times 6 plus the total number of behinds scored (by all teams):

x * number of games = 6 * total number of goals + total number of behinds

Substituting the first equation into the second equation, we get:

(6a/13 + b/15) * number of games = 6 * total number of goals + total number of behinds

Simplifying this equation and solving for "number of games", we get:

number of games = 1170/(2a/13 + b/15)

Since the number of games must be an integer, we can see that 2a/13 + b/15 must be a divisor of 1170. The possible values of 2a/13 + b/15 are:

2/13 + 78/15 = 72/5

4/13 + 72/15 = 56/5

6/13 + 66/15 = 44/5

8/13 + 60/15 = 32/5

The only divisor of 1170 among these values is 72/5, which corresponds to a = 26 and b = 312. Therefore, the number of games played in the round is:

number of games = 1170 / [(2a/13) + (b/15)]

                              = 1170 / [(2*26/13) + (312/15)]

                              = 195

As a result, 195 games have been played in the competition.

To know more about the Game, here

https://brainly.com/question/27890331

#SPJ4

a survey reports that 67% of college students prefer to drink more coffee during the exams week. if we randomly select 80 college students and ask each whether they drink more coffee during exams week. what is the probability that at most 60 say that they drink coffee during exam week?

Answers

From that 80 college students, the probability that at most 60 say that they drink coffee during exam week  is 0.3085

This is a binomial distribution problem in which we want to know the probability it is that at least 60 students out of 80 choose to drink coffee during test week.

Here, we have:

n = 80 (number of trials)

p = 0.67 (probability of success in each trial)

q = 1 - p = 0.33 (probability of failure in each trial)

x ≤ 60 (number of successes we want to find the probability for)

This probability may be calculated using the binomial cumulative distribution function (CDF). The binomial CDF formula is as follows:

P(X ≤ k) = Σi=[tex]0^{K}[/tex] ([tex]_{n}C^{i} }[/tex]) * [tex]p^{i}[/tex] * ([tex](1-p)^{n-i}[/tex]

We can determine the chance of having 60 or fewer successes using this formula:

P(X ≤ 60) = Σi=[tex]0^{60}[/tex] ([tex]_{80} C^{i}[/tex]) * [tex]0.67^{i}[/tex] * [tex]0.33^{80-i}[/tex]

P(X ≤ 60) = 0.3085

As a result, the probability that at least 60 college students claim they consume coffee during test week is 0.3085, or around 31%. As a result, 0.3085 is the correct answer.

Learn more about Binomial Cumulative Distribution Function (CDF):

https://brainly.com/question/30487626

#SPJ4

Can someone help me asap? It’s due tomorrow. I will give brainiest if it’s correct. Provide an explanation

Answers

We can expect approximately 144 students in Chloe's school to own pets. The answer is 144.

What is mean by Proportion ?

A proportion is an equation in which two ratios are set equal to each other. For example, if there is 1 boy and 3 girls you could write the ratio as: 1 : 3 (for every one boy there are 3 girls) 1 / 4 are boys and 3 / 4 are girls. 0.25 are boys (by dividing 1 by 4)

Based on the simulation results, we can count the number of times the number cube landed on digits 1, 2, 3, and 4, which represent the number of students who own pets. We add up these counts and divide by the total number of rolls (25) to get the proportion of rolls that resulted in a student owning a pet:

(5+1+4+4+3+3+2+2+2+1+5+2+1) / 25 = 0.6

So, approximately 60% of the rolls resulted in a student owning a pet. If we assume this proportion holds for the entire school, we can estimate the number of students who own pets out of the total number of students in the school:

Number of students who own pets = Proportion of students who own pets * Total number of students

Number of students who own pets = 0.6 * 240 = 144

Therefore, we can expect approximately 144 students in Chloe's school to own pets. The answer is 144.

Learn more about ratios here

https://brainly.com/question/13419413

#SPJ1

the rule T(-3,1) is applied to point 2,-7 in which part of the coordinate system is the translated point

Answers

the translated point is located in the third quadrant of the coordinate system, since both coordinates are negative.

What is Cartesian coordinate?

A coordinate system, also known as a Cartesian coordinate system, is a system used to describe the position of points in space. It is named after the French mathematician and philosopher René Descartes, who introduced the concept in the 17th century. In a coordinate system, each point is assigned a unique pair of numbers, called coordinates, that describe its position relative to two perpendicular lines, called axes. The horizontal axis is usually labeled x and the vertical axis is usually labeled y.

To apply the translation rule T(-3, 1) to the point (2, -7), we need to add the translation vector (-3, 1) to the coordinates of the point:

(2, -7) + (-3, 1) = (-1, -6)

The resulting point after the translation is (-1, -6).

Therefore, the translated point is located in the third quadrant of the coordinate system, since both coordinates are negative.

Learn more about Cartesian coordinates, by the following link

https://brainly.com/question/17206319

#SPJ9

What is the answer to this problem?

Answers

The area of the shaded area is 3.27 ft²

How to the area of the shaded area?

We can find the area of the shaded area by subtracting the area of the triangle from the area of the sector. That is;

Area of shaded area = Area of sector - area of triangle

Area of shaded area = (60/360 * π * 6²) - (1/2 * 6 * 6 * sin 60)

Area of shaded area = (60/360 * 22/7 * 36) - (1/2 * 36 * 0.866)

Area of shaded area = 3.27 ft²

Learn more about shaded area on:

https://brainly.com/question/14989383

#SPJ1

when utilizing a matlab built-in ode solver, in the function for one's states the variable for the state derivatives must be organized as a column vector. group of answer choices true false

Answers

It is true to say that when utilizing MATLAB's built-in ode solver, the function for one's states the variable for the state derivatives must be set as column vectors.

The Ordinary Differential Equation (ODE) solvers in MATLAB solve initial value problems with a varient state of properties. They consider state spaces as a column vector. For example, two solve a second degree ODE, It's two states in state space has to be arranges in a 1x2 column vector to pass it into a ODE solver.

Initial value problems with different attributes can be solved using the

ODE solvers in MATLAB.

Here the first ODE is of third order, so it will be converted to three equivalent first order ODEs. The second ODE is second order, so it will be converted to two equivalent first order ODEs.These solver can be used to solve the different type like the differential algebraic equations (DAEs), problems involving a mass matrix, and fully implicit problems. ODE45, is most frequently used ODE solver in MATLAB.

It used to compare methods of orders four and five to determine an estimate error.

For more information about matlab, visit :

https://brainly.com/question/30479968

#SPJ4

Exit
6-3: PRACTICE Part 2 Logarithms in Equations Algebra 2
Michael invests $1,000 in an account that earns a 4.75% annual percentage rate compounded continuously. Peter invests $1,200 in an account that earns a 4.25% annual
percentage rate compounded continuously. Which person's account will grow to $1,800 first?
Michael's account will grow to $1,800 after about year(s). Peter's account will grow to $1,800 after about year(s). So,
(Round to the nearest whole number as needed.)
Video Textbook Get more help.
Review Progress
√₁
Vi
More
Question
8
Clear all
of 9
Due 04/13/23
account will grow to $1,800 first.
Check answer
< Back
Next

Answers

Answer: To solve this problem, we need to use the continuous compound interest formula:

A = Pe^(rt)

where A is the amount in the account, P is the initial principal, e is the mathematical constant e (approximately 2.71828), r is the annual interest rate (as a decimal), and t is the time in years.

For Michael's account, we have:

A = 1000e^(0.0475t)

For Peter's account, we have:

A = 1200e^(0.0425t)

We want to find the time it takes for each account to reach $1,800. So we can set up the following equations:

1000e^(0.0475t) = 1800

1200e^(0.0425t) = 1800

We can solve each equation for t by taking the natural logarithm of both sides and isolating t:

ln(1000) + 0.0475t = ln(1800)

ln(1200) + 0.0425t = ln(1800)

Subtracting ln(1000) or ln(1200) from both sides, we get:

0.0475t = ln(1800) - ln(1000)

0.0425t = ln(1800) - ln(1200)

Dividing both sides by the interest rate and simplifying, we get:

t = (ln(1800) - ln(1000)) / 0.0475 ≈ 10.16 years for Michael's account

t = (ln(1800) - ln(1200)) / 0.0425 ≈ 10.62 years for Peter's account

Therefore, Michael's account will grow to $1,800 first, after about 10 years (rounded to the nearest whole number).

Step-by-step explanation:

I don’t know the answer to the math problem

Answers

Answer:

54.1 cm

Step-by-step explanation:

1) Divide the circumference by pi (3.14…)

2) Your quotient (108.2… cm) is now the diameter of the circle. The diameter goes through the circle completely. The radius is half of the diameter, or it goes halfway through the circle. Hence, you would divide the previous answer by 2.

3) 108.2… cm divided by 2 should leave you with about 54.1… cm.

PLEASE HELPPPPP!!!!! Which statement correctly compares the shapes of the of the distributions!

Answers

Answer:

Could be B

Step-by-step explanation:

Southview HS is mirrored or symmetrical while the other is going up.

three bolts and three nuts are in a box. two parts are chosen at random. find the probability that one is a bolt and one is a nut.

Answers

The probability of picking one bolt and one nut is 1/2 or 50%.

To find the probability that one is a bolt and one is a nut, we need to use the formula for calculating the probability of two independent events happening together: P(A and B) = P(A) × P(B)

Let's first calculate the probability of picking a bolt from the box:

P(bolt) = number of bolts / total number of parts = 3/6 = 1/2

Now, let's calculate the probability of picking a nut from the box:

P(nut) = number of nuts / total number of parts = 3/6 = 1/2

Since the events are independent, the probability of picking a bolt and a nut in any order is:

P(bolt and nut) = P(bolt) × P(nut) + P(nut) × P(bolt)

P(bolt and nut) = (1/2) × (1/2) + (1/2) × (1/2)

P(bolt and nut) = 1/2

Therefore, the probability of picking one bolt and one nut is 1/2 or 50%.

Learn more about  probability

https://brainly.com/question/30034780

#SPJ4

the probability that one chosen part is a bolt and the other chosen part is a nut is 1, or 100%. This makes sense because if we choose two parts at random, we must get one bolt and one nut since there are three of each in the box.

To find the probability that one chosen part is a bolt and the other chosen part is a nut, we need to use the formula for probability:

Probability = (number of desired outcomes) / (total number of outcomes)

There are two ways we could choose one bolt and one nut: we could choose a bolt first and a nut second, or we could choose a nut first and a bolt second. Each of these choices corresponds to one desired outcome.

To find the number of ways to choose a bolt first and a nut second, we multiply the number of bolts (3) by the number of nuts (3), since there are 3 possible bolts and 3 possible nuts to choose from. This gives us 3 x 3 = 9 total outcomes.

Similarly, there are 3 x 3 = 9 total outcomes if we choose a nut first and a bolt second.

Therefore, the total number of desired outcomes is 9 + 9 = 18.

The total number of possible outcomes is the number of ways we could choose two parts from the box, which is the number of ways to choose 2 items from a set of 6 items. This is given by the formula:

Total outcomes = (6 choose 2) = (6! / (2! * 4!)) = 15

Putting it all together, we have:

Probability = (number of desired outcomes) / (total number of outcomes)
Probability = 18 / 15
Probability = 1.2

However, this answer doesn't make sense because probabilities should always be between 0 and 1. So we made a mistake somewhere. The mistake is that we double-counted some outcomes. For example, if we choose a bolt first and a nut second, this is the same as choosing a nut first and a bolt second, so we shouldn't count it twice.

To correct for this, we need to subtract the number of outcomes we double-counted. There are 3 outcomes that we double-counted: choosing two bolts, choosing two nuts, and choosing the same part twice (e.g. choosing the same bolt twice). So we need to subtract 3 from the total number of desired outcomes:

Number of desired outcomes = 18 - 3 = 15

Now we can calculate the correct probability:

Probability = (number of desired outcomes) / (total number of outcomes)
Probability = 15 / 15
Probability = 1

So the probability that one chosen part is a bolt and the other chosen part is a nut is 1, or 100%. This makes sense because if we choose two parts at random, we must get one bolt and one nut since there are three of each in the box.

learn more about probability

https://brainly.com/question/30034780

#SPJ11

help would be absolutely appreciated

Answers

Thus, the given quadratic equation has two real roots x = 3 and x = -2.

Explain about the solution of quadratic function:

A function or mathematical statement of degree two is a quadratic function. This indicates that two is the function's highest power. The roots of all quadratic functions are two.

The quadratic formula is employed to solve a quadratic to discover its roots, which can either be distinct or the same. A quadratic function must first be converted into a quadratic equation by being made equal to zero in order to be solved.

Given equation:

x² - x - 6 = 0

Using the quadratic formula:

x = [-b ± √(b² - 4ac) ] / 2a

a = 1 , b = -1 and c = -6

x = [1 ± √((-1)² - 4*1*(-6) ] / 2*1

x = [1 ± √(1 + 24) ] / 2

x = [1 ± 5 ] / 2

Now,

x = (1 + 5)/2 = 3

x = (1 - 5)/ 2 = -2

Thus, the given quadratic equation has two real roots x = 3 and x = -2.

Know more about the quadratic function:

https://brainly.com/question/1214333

#SPJ1

complete question:

Check whether the given equation has, one solution, two solution ,many solution or no real solution.

x² - x - 6 = 0

Given that £1 = $1.62
a) How much is £650 in $?
b) How much is $405 in £?

Answers

Answer:

a 1053

b 250

multiply 650 by 1.62 for part a.

for part b divide by 1.62 since pound is less than dollar

hope this helps :)

PLEASE HELP!! 20 POINTS

Select the correct answer from each drop-down menu. Consider this equation. 1/x + 2/x+10 =1/3 Complete the statements to make them true. The least common denominator is . The equation will have valid solutions.

Answers

Answer: The least common denominator is 3x(x+10)

The equation will have 2 valid solutions

Step-by-step explanation: sorry if wrong

Part B


Based on your construction, what do you know about ΔABD and ΔBCD?

Answers

The construction and the resulting triangles are interesting because they allow us to explore the properties of perpendicular lines and the angles they form.

Now, let's look at the two triangles that are formed as a result of this construction - ΔABD and ΔBCD. Since line BD is perpendicular to line AC, we know that angle ABD and angle CBD are both right angles. This is because any line that is perpendicular to another line forms a right angle with that line.

Now, let's look at the other sides of the triangles. In ΔABD, we have side AB, which is different from side BC in ΔBCD. Similarly, in ΔBCD, we have side CD, which is different from side AD in ΔABD.

So, although the two triangles share a common side (BD), they have different lengths for their other sides. This means that the two triangles are not congruent, since congruent triangles must have the same length for all their sides.

However, we can still find some similarities between the two triangles. For example, since angle ABD and angle CBD are both right angles, we know that they are congruent. Additionally, we can use the fact that angle ADB is congruent to angle CDB, since they are alternate interior angles formed by a transversal (line BD) intersecting two parallel lines (line AC and the line perpendicular to it passing through point B).

To know more about triangle here

https://brainly.com/question/8587906

#SPJ4

Complete Question:

Draw a line through point B that is perpendicular to line AC Label the intersection of the line and line AC as point D. Take a screenshot of your work, save it, and insert the image in the space below.

Part B

Based on your construction, what do you know about ΔABD and ΔBCD?

Select the GCF of these numbers. 48 and 60 22 ·3 2· 112 32 23 · 5 13· 193 ·232

Answers

The GCF of 48 and 60 is 12

To find the greatest common factor (GCF) of 48 and 60, we can start by finding the prime factorization of each number

48 = 2^4 × 3

60 = 2^2 × 3 × 5

Next, we can identify the common factors of both numbers by looking at their prime factorization

The factors of 48 are: 1, 2, 3, 4, 6, 8, 12, 16, 24, 48

The factors of 60 are: 1, 2, 3, 4, 5, 6, 10, 12, 15, 20, 30, 60

The common factors of 48 and 60 are: 1, 2, 3, 4, 6, and 12.

The greatest common factor is the largest number that both 48 and 60 can be divided evenly by. In this case, that number is 12. Therefore, the GCF is 12.

Learn more about greatest common factor here

brainly.com/question/11320970

#SPJ4

The given question is incomplete, the complete question is:

Find the GCF of 48 and 60.


The function S(d) = √9.8d estimates the speed, S, in meters/second (m/s),
of a tsunami based on the ocean depth, d, in meters (m).
Determine the speed, in (m/s), of a tsunami at the depth of 2145 m. Round your
answer to the nearest thousandth.

Answers

To find the speed of the tsunami at the depth of 2145 meters, you can use the given function:

S(d) = √(9.8 * d)

Plug in the value of d (2145 meters):

S(2145) = √(9.8 * 2145)S(2145) ≈ √(21001)S(2145) ≈ 144.917Rounded to the nearest thousandth, the speed of the tsunami at a depth of 2145 meters is approximately 144.917 m/s.

an agency has specialists who analyze the frequency of letters of the alphabet in an attempt to decipher intercepted messages. suppose a particular letter is used at a rate of 6.6%. what is the mean number of times this letter will be found on a typical page of 2650 characters? 174.9 what is the standard deviation for the number of times this letter will be found on a typical page of 2650 characters ? round your answer to 1 decimal place. in an intercepted message, a page of 2650 characters is found to have the letter occurring192 times. would you consider this unusual?

Answers

Standard deviation normal distribution table or calculator to determine the probability of observing a z-score of 1.3 or higher.

The probability is approximately 0.0968, or 9.68%.

To determine the mean number of times the letter appears on a page, we can multiply the probability of the letter appearing (0.066) by the total number of characters on the page (2650):

[tex]Mean = 0.066 \times 2650 = 174.9[/tex]

To calculate the standard deviation, we can use the formula:

Standard deviation = [tex]\sqrt(n \times p \times q)[/tex]

n is the sample size (2650), p is the probability of success (0.066), and q is the probability of failure [tex](1 - p = 0.934)[/tex].

Standard deviation = [tex]sqrt(2650 \times 0.066 \times 0.934) = 13.2[/tex] (rounded to 1 decimal place)

Determine whether 192 occurrences of the letter on a page is unusual, we can use the z-score formula:

z = (x - mean) / standard deviation

x is the observed number of occurrences (192), mean is the expected number of occurrences (174.9), and standard deviation is the standard deviation we just calculated (13.2).

[tex]z = (192 - 174.9) / 13.2 = 1.3[/tex]

For similar questions on Standard deviation

https://brainly.com/question/475676

#SPJ11

what is the maximum number of consecutive odd positive integers that can be added together before the sum exceeds ?

Answers

The maximum number of consecutive odd positive integers that can be added together before the sum exceeds 401 is 11.

Let's assume the first odd integer is x. Then, the sum of the next n consecutive odd integers would be given by:

x + (x+2) + (x+4) + ... + (x+2n-2) = nx + 2(1+2+...+n-1) = nx + n(n-1)

We want to find the largest n such that the sum is less than or equal to 401:

nx + n(n-1) ≤ 401

Since the integers are positive and odd, we can start with x=1 and then try increasing values of n until we find the largest value that satisfies the inequality:

n + n(n-1) ≤ 401

n² - n - 401 ≤ 0

Using the quadratic formula, we find that the solutions are:

n = (1 ± √(1+1604))/2

n ≈ -31.77 or n ≈ 32.77

We discard the negative solution and round down to the nearest integer, giving us n = 11. Therefore, the maximum number of consecutive odd positive integers that can be added together before the sum exceeds 401 is 11.

Learn more about sum

https://brainly.com/question/24205483

#SPJ4

Complete Question:

what is the maximum number of consecutive odd positive integers that can be added together before the sum exceeds 401?

Kubin Company’s relevant range of production is 25,000 to 33,500 units. When it produces and sells 29,250 units, its average costs per unit are as follows: Average Cost per Unit Direct materials $ 8. 50 Direct labor $ 5. 50 Variable manufacturing overhead $ 3. 00 Fixed manufacturing overhead $ 6. 50 Fixed selling expense $ 5. 00 Fixed administrative expense $ 4. 00 Sales commissions $ 2. 50 Variable administrative expense $ 2. 00 Required: 1. For financial accounting purposes, what is the total amount of product costs incurred to make 29,250 units? 2. For financial accounting purposes, what is the total amount of period costs incurred to sell 29,250 units? 3. For financial accounting purposes, what is the total amount of product costs incurred to make 33,500 units? 4. For financial accounting purposes, what is the total amount of period costs incurred to sell 25,000 units? (For all requirements, do not round intermediate calculations. )


1. Total amount of product costs

2. Total amount of period costs incurred

3. Total amount of product costs

4. Total amount of period costs

Answers

For the relevant range of production of units total amount of product and period cost as per units are,

Total amount of product costs for 29,250 units is $687,375.

Total amount of period costs incurred for 29,250 units is $58,511.50

Total amount of product costs for 33,500 units is equal to $787,250.

Total amount of period costs for 25,000 units  is equal to $50,011.50.

Average Cost per Unit Direct materials  = $ 8. 50

Direct labor = $ 5. 50

Variable manufacturing overhead = $ 3. 00

Fixed manufacturing overhead = $ 6. 50

Fixed selling expense  = $ 5. 00

Fixed administrative expense = $ 4. 00

Sales commissions = $ 2. 50

Variable administrative expense = $ 2. 00

Total unit produced = 29,250 units,

Total product costs

= (Direct materials + Direct labor + Variable manufacturing overhead + Fixed manufacturing overhead) x Number of units produced

= ($8.50 + $5.50 + $3.00 + $6.50) x 29,250

= $23.50 x 29,250

= $687,375

The total amount of product costs incurred to make 29,250 units is $687,375.

Total period costs

= Fixed selling expense + Fixed administrative expense + Sales commissions + (Variable administrative expense x Number of units sold)

= $5.00 + $4.00 + $2.50 + ($2.00 x 29,250)

= $5.00 + $4.00 + $2.50 + $58,500

= $58,511.50

The total amount of period costs incurred to sell 29,250 units is $58,511.50

For the number of units produced changed to 33,500.

Total product costs

= (Direct materials + Direct labor + Variable manufacturing overhead + Fixed manufacturing overhead) x Number of units produced

= ($8.50 + $5.50 + $3.00 + $6.50) x 33,500

= $23.50 x 33,500

= $787,250

The total amount of product costs incurred to make 33,500 units is $787,250.

The number of units sold changed to 25,000.

Total period costs

= Fixed selling expense + Fixed administrative expense + Sales commissions + (Variable administrative expense x Number of units sold)

= $5.00 + $4.00 + $2.50 + ($2.00 x 25,000)

= $5.00 + $4.00 + $2.50 + $50,000

= $50,011.50

The total amount of period costs incurred to sell 25,000 units is $50,011.50.

Therefore, the total amount of the product and period cost for different situations are,

Total amount of product costs is equal to $687,375.

Total amount of period costs incurred is equal to $58,511.50

Total amount of product costs is equal to $787,250.

Total amount of period costs is equal to $50,011.50.

learn more about cost here

brainly.com/question/17927959

#SPJ4

HURRY 40 POINTS!!

What is the surface area of this right rectangular prism?

Enter your answer as a mixed number in simplest form by filling in the boxes.

ft²

Answers

The surface area of the rectangular prism is 29 2/3 ft²

How to determine the surface area

The formula for calculating the surface area of a rectangular prism is expressed as;

SA = 2(wl + hw + hl)

Where the parameters are;

SA is the surface areaw is the width of the prismh is the height of the prisml is the length of the prism

From the information given, we have that;

Wl = 3 × 5/2

multiply the values

wl = 15/2

hw = 4/3 × 3

hw = 4

hl = 4/3 × 5/2 = 20/6 = 10/3

Substitute the values

Surface area = 2(4 + 10/3 + 15/2)

Surface area = 2(24 + 20 + 45/6)

Surface area = 2(89)/6

Surface area = 89/3 = 29 2/3 ft²

Learn about surface area at: https://brainly.com/question/24284033

#SPJ1

A student is helping a family member build a storage bin for their garage. They would like for the bin to have a volume of 240 ft3 If they already have the length measured at 8 feet and the width at 6 feet, what is the height needed to reach the desired volume?

(A) 3 feet
(B) 3.5
(C) 4 feet
(D) 5 feet​

Answers

Answer: The answer to your question is D! Brainliest?

Step-by-step explanation:

To find the height needed to reach a volume of 240 ft^3, we can use the formula:

Volume = length x width x height

Substituting the given values, we get:

240 = 8 x 6 x height

Simplifying:

240 = 48 x height

height = 240/48

height = 5

Therefore, the height needed to reach a volume of 240 ft^3 is 5 feet.

Answer: (D) 5 feet.

Each year the school randomly selects a sporting event for their family fun night. The superintendent also randomly selects a sporting event to attend. What is the probability that the superintendent will be at the family fun night sporting event?

Answers

The probability that the superintendent will be at the family fun night sporting event is 1/n, where n is the total number of sporting events held by the school each year.

what is probability?

Probability is a measure of the likelihood of an event occurring. It is a number between 0 and 1, where 0 means the event is impossible and 1 means the event is certain to happen.

Assuming that the superintendent is equally likely to attend any of the school's sporting events, the probability that they attend the family fun night sporting event is simply the ratio of the number of family fun night sporting events to the total number of sporting events.

If we assume that the school has n sporting events each year and that the family fun night sporting event is chosen randomly from these n events, then the probability that the superintendent attends the family fun night sporting event is 1/n.

Therefore, the probability that the superintendent will be at the family fun night sporting event is 1/n, where n is the total number of sporting events held by the school each year.

To learn more about probability from the given link:

https://brainly.com/question/30034780

#SPJ1

Need help with this!

Answers

Answer

Number 14: 7 faces, 15 edges, and 10 vertices.

Number 15: 10 faces, 24 edges, and 16 vertices.

Number 16: 7 faces, 12 edges, and 7 vertices.

:D

Step-by-step explanation:

Shopping While shopping for clothes, Tracy spent $34 less than 3 times what Jaclyn spent. Tracy spent $26. Write and solve an equation to find how much Jaclyn spent. Let x represent how much Jaclyn spent. The equation that can be used to determine how much Jaclyn has spent is

Answers

Let x be the amount that Jaclyn spent on clothes.

According to the problem, Tracy spent $34 less than 3 times what Jaclyn spent, which can be expressed as:

3x - 34

And we also know that Tracy spent $26. So we can set up an equation:

3x - 34 = 26

To solve for x, we can add 34 to both sides of the equation:

3x = 60

And then divide both sides by 3:

x = 20

Therefore, Jaclyn spent $20 on clothes.

The area of a rectangle is 8811m if the width of the garden is 89 m what’s the length

Answers

The length of the garden is 99 m.

What’s the length?

The formula for the area of a rectangle is:

Area = Length x Width

We are given that the area of the rectangle is 8811 [tex]m^{2}[/tex] and the width is 89 m. Substituting these values into the formula, we get:

8811 [tex]m^{2}[/tex] = Length x 89 m

To solve for the length, we can divide both sides of the equation by 89 m:

Length = 8811 [tex]m^{2}[/tex] / 89 m

Simplifying, we get:

Length = 99 m

Therefore, the length of the garden is 99 m.

to know more about rectangle

brainly.com/question/29123947

#SPJ1

A large rectangular prism is 5 feet long, 3 feet wide, and 4 feet tall. A small rectangular prism is 2.5 feet long, 1.5 feet wide, and 2 feet tall.
How many small prisms would it take to fill the large prism?
Write your answer as a whole number or decimal. Do not round.

Answers

The answer of the given question based on the  rectangular prism is , , it would take 8 small rectangular prisms to fill the large rectangular prism.

What is Rectangular prism?

A rectangular prism, also known as a rectangular parallelepiped, is a three-dimensional solid object that has six rectangular faces, with opposite faces being congruent and parallel. It is a special case of a parallelepiped in which all angles are right angles and all six faces are rectangles.

To find how many small rectangular prisms will fit inside the large rectangular prism, we need to calculate the volume of each prism and then divide the volume of the large prism by the volume of the small prism.

The volume of the large prism is:

V_large = length × width × height = 5 ft × 3 ft × 4 ft = 60  feet³

The volume of the small prism is:

V_small = length × width × height = 2.5 ft × 1.5 ft × 2 ft = 7.5  feet³

Dividing the volume of the large prism by the volume of the small prism, we get:

number of small prisms = V_large / V_small = 60 ft³ / 7.5 ft³ = 8

Therefore, it would take 8 small rectangular prisms to fill the large rectangular prism.

To know more about Volume visit:

https://brainly.com/question/29255732

#SPJ1

Which of the following best describes the expression 9(x + 7)? (20 brainly points)
A: The sum of constant factors 9 and x + 7
B: The product of constant factors 9 and x + 7
C: The product of a constant factor 9 and a 2-term factor x + 7
D: The sum of a constant factor 9 and a 2-term factor x + 7

Answers

The statement that express 9(x + 7) is product of constant factors 9 and x + 7. The Option B.

What does the expression 9(x + 7) represent?

The expression 9(x + 7) represents the product of constant factors 9 and x + 7. To evaluate the expression, you would distribute the 9 to both terms inside the parentheses, resulting in 9x + 63.

This expression can also be written as a 2-term factor of 9 and x + 7. It is important to understand the different terms and factors in an expression to simplify and solve equations.

Read more about product

brainly.com/question/13152087

#SPJ1

The radius of a circle is 10 cm. Find its area in terms of π.

Answers

The area of a circle is given by the formula:

A = πr^2

where r is the radius of the circle.

Substituting the value of the radius as r = 10 cm, we get:

A = π(10)^2

A = 100π

Therefore, the area of the circle with radius 10 cm is 100π square centimeters.

~~~Harsha~~~

A drawer contains 3 red paper clips, 4 green paper clips, and 5 blue paper clips. One paper clip is taken from the drawer and is NOT replaced. Another paper clip is taken from the drawer. What is the probability that the first paper clip is red and the second paper clip is blue?

Answers

The probability of drawing a red paper clip first and a blue paper clip second is 15/132 or approximately 0.1136.

What is probability?

Probability is a measure of the likelihood of an event occurring. It is a number between 0 and 1, where 0 means the event is impossible and 1 means the event is certain to happen.

The probability of drawing a red paper clip on the first try is 3/12, because there are 3 red paper clips out of 12 total paper clips in the drawer. Once a paper clip has been removed, there are 11 paper clips remaining, so the probability of drawing a blue paper clip on the second try is 5/11, because there are 5 blue paper clips remaining out of the 11 total remaining paper clips.

The probability of both of these events occurring is the product of their individual probabilities, so the probability of drawing a red paper clip first and a blue paper clip second is:

(3/12) * (5/11) = 15/132

Therefore, the probability of drawing a red paper clip first and a blue paper clip second is 15/132 or approximately 0.1136.

To learn more about probability from the given link:

https://brainly.com/question/30034780

#SPJ1

What is the equation of the line in slope-intercept form?

Answers

Answer:

y = 3/5x + 3

Step-by-step explanation:

points on the graph

(-5,0) and (0,3)


0- 3 = -3

-5 - 0 = -5

-3/-5= 3/5


y = 3/5x + B

use a point from the graph

3 = 3/5 x 0 + B

3 = 0 + B

3 -0 = 3

3 = B

check answer

(-5,0)

Y = 3/5 x -5 + 3

Y = -15/3 + 3

Y = -3 + 3

Y = 0

Making the equation true y = 3/5x + 3

Other Questions
The tire had an initial volume of 7 liters, at a temperature of 25 C. After driving for an hour, friction from the road had increased the temperature of air in the tire to 35 C. Assuming the pressure inside the tire did not change, what would the tires new volume be? Which of the following is true?Principals are not usually specified in a currency swapThe principal amounts usually flow in the opposite direction to interest payments at the beginning of a currency swap and in the same direction as interest payments at the end of the swap.The principal amounts usually flow in the same direction as interest payments at the beginning of a currency swap and in the opposite direction to interest payments at the end of the swap.Principals are not usually exchanged in a currency swap What are some disadvantages of Partnerships in Foreign Direct Investments? (Try to go into good detail explaining the disadvantages) (also make sure you talk about the disadvantages with PARTNERSHIPS in FDI!) 1.) Caspian Sea Drinks is considering the purchase of a new water filtration system produced by Rube Goldberg Machines. This new equipment, the RGM-7000, will allow Caspian Sea Drinks to expand production. It will cost $12.00 million fully installed and will be fully depreciated over a 19.00 year life, then removed for no cost. The RGM-7000 will result in additional revenues of $3.17 million per year and increased operating costs of $729,643.00 per year. Caspian Sea Drinks' marginal tax rate is 29.00%. The incremental cash flows for produced by the RGM-7000 are _____. some critics argue that women are not fit to be in leadership positions because of their gender. which perspective are these critics exemplifying? sociology with common control wiring, the power source for the control circuit is separate from the power circuit wiring. true or false? an adult is diagnosed with communicating hydrocephalus. the form of hydrocephalus in adults is most often caused by: Which of the following is a focus of the APA Professional Code of Conduct for Psychologists? Which source is a secondary source? (1 point)O a photographO a magazine articleO a diaryO a history book a. The circumference of the base of a cylinder is 82.37cm and its height is 21.20 cm. Find the curved surface area the volume of a cylinder b. Draw a table for multiplication, in modulo base 7 on the set X =357 Between which two integers does 7/3 lie Answer A 4 and 5 Answer B 1 and 2 Answer C 2 and 3 Answer D 3 and 4 felicia was fired from her job. she has not found a new one. she wants to retain her health insurance that was provided by her employer who fired her. is this possible? In the coordinate plane, a square has vertices (4, 3), (-3, 3), (-3, - 4). What is the location of the fourth vertex? You are considering how to invest part of your retirement savings.You have decided to put $400,000into three stocks:52% of the money in GoldFinger (currently $15/share), 19% of the money in Moosehead (currently $90/share), and the remainder in Venture Associates (currently $2/share). Suppose GoldFinger stock goes up to $33/share, Moosehead stock drops to $59/share, and Venture Associates stock rises to $11 per share.a. What is the new value of the portfolio?b. What return did the portfolio earn?c. If you don't buy or sell any shares after the price change, what are your new portfolio weights?Question content area bottomPart 1a. What is the new value of the portfolio?The new value of the portfolio is$enter your response here. (Round to the nearest dollar.)Part 2b. What return did the portfolio earn?The portfolio earned a return ofenter your response here%.(Round to two decimal places.)Part 3c. If you don't buy or sell any shares after the price change, what are your new portfolio weights?The weight of Goldfinger is nowenter your response here%.(Round to two decimal places.)Part 4The weight of Moosehead is nowenter your response here%.(Round to two decimal places.)Part 5The weight of Venture is nowenter your response here%. plasma angiotensin ii levels would be higher when mean arterial blood pressure is ______. A company is going to make an oil container in the shape of a cylinder. As shown below, the container will have a height of 8 m and a diameter of 10 m. The container will be made from steel (including its top and bottom). Suppose the total cost of the steel will be $13,062.40. How much will the steel cost per square meter? Use 3.14 for it, and do not round your answer. what constant acceleration is required to increase the speed of a car from 22 mi/h to 58 mi/h in 2 s? (round your answer to two decimal places.) a company has multiple cas and intermediate cas issuing digital certificates in different departments, with no one cross-checking their work. which pki trust model should the company use? a. bridge trust model b. web of trust model c. hierarchical trust model d. distributed trust model T or FAccording to the latest postmodern theory, change begins when clients focus on present problems rather than future solutions Determine which relation is a function. A: {(3, 2), (1, 3), (1, 2), (0, 4), (1, 1)} B: {(3, 2), (2, 3), (1, 1), (0, 4), (0, 1)} C: {(3, 3), (2, 3), (1, 1), (0, 4), (0, 1)} D: {(3, 2), (2, 3), (1, 2), (0, 4), (1, 1)}